write an expression for the apparent nth term of the sequence.
(assume that n begins with 1.)
-243,729,-2187,6561,-19683,...

Answers

Answer 1

The given sequence -243, 729, -2187, 6561, -19683, ... can be expressed by the apparent nth term as (-3)^n.

The given sequence appears to be a geometric sequence with a common ratio of -3. To find the apparent nth term, we can express it using the general formula for a geometric sequence.

The formula for the nth term of a geometric sequence is given by:

an = a1 * r^(n-1)

Where an represents the nth term, a1 is the first term, r is the common ratio, and n is the position of the term in the sequence.

In this case, the first term a1 is -243 and the common ratio r is -3. Substituting these values into the formula, we get:

an = -243 * (-3)^(n-1)

Therefore, the apparent nth term of the given sequence is -243 * (-3)^(n-1).

Learn more about sequence here : brainly.com/question/23857849

#SPJ11


Related Questions

please help me with this question

Answers

The correct simplified form of the expression is x + 5.

a) The mistake that Hannah has made is incorrectly combining the terms 3x and -2x. Instead of subtracting the coefficients of x, she subtracted the entire expression 2x from 3x.

b) To simplify the expression correctly, we need to combine like terms. In this case, the like terms are the ones with the variable x.

The expression 3x + 5 - 2x can be rewritten as (-2x + 3x) + 5.

Now, let's combine the like terms:

(-2x + 3x) + 5 = x + 5

Therefore, the correct simplified form of the expression is x + 5.

To further clarify, Hannah mistakenly thought that subtracting 2x from 3x would result in 1x (or just x). However, when subtracting or adding terms with the same variable, we need to consider the coefficients. In this case, 3x - 2x simplifies to x, not 1x.

It's important to pay attention to the signs and operations when combining terms. In this scenario, Hannah overlooked the need to subtract the coefficients of x and ended up with an incorrect result.

For more such questions on expression visit:

https://brainly.com/question/1859113

#SPJ8

compute the winner of each match is the team who has the highest
score. the team that is winner scores 3 and 1 point for a draw and
the team with the most points at the end of the season is the
winner

Answers

It is clear that the team that has accumulated the most points by the end of the season is declared the winner.

The winner of each match is the team who has the highest score. The team that is the winner scores 3 points, and 1 point is for a draw. The team with the most points at the end of the season is the winner. The league system is a format in which teams compete against each other in a regular season, with the team with the most points being crowned the winner at the end of the season.

When two teams compete against each other in a match, the winner of the match is the team that has the most points at the end of the match.

This typically means that the team with the most goals is the winner, although some leagues may use other criteria to determine the winner, such as the number of corners, free kicks, or other statistical measures . For each win, a team gets three points. In a case where both teams score the same number of goals, the match ends in a draw, and each team receives one point.

For example, let us assume that Team A won 10 matches, drew three, and lost five matches. If Team B won eight matches, drew five, and lost five matches, Team A would be declared the winner because they had 33 points (10 x 3 points for a win + 3 x 1 point for a draw), while Team B had only 29 points (8 x 3 points for a win + 5 x 1 point for a draw).

Therefore, it is clear that the team that has accumulated the most points by the end of the season is declared the winner.

To know more about Winner  visit :

https://brainly.com/question/30135829

#SPJ11

Which is a zero of the function f(x)= x^ - 3x - 4

a) -4
b) -1
c) 0
d) 1

Answers

Answer:

B) -1

Step-by-step explanation:

[tex]f(x)=x^2-3x-4\\0=x^2-3x-4\\0=(x-4)(x+1)\\x=4,-1[/tex]

Therefore, B is the best choice

EXAMPLE 2 The set {sint, cost} is linearly independent in C[0, 1], the space of all continuous functions on 0 ≤ t ≤ 1. Explain why?

Answers

The set {sine of t, cosine of t} forms a linearly independent set in C[0, 1] due to the fact that the sole method of representing the zero function as a linear combination of sine of t and cosine of t is by assigning a value of zero to each coefficient.

Why the set is linearly independent?

To prove this, we suppose that there exist constants a and b such that;

[tex]a sin t + b cos t = 0[/tex]

For t in [0, 1]. We can differentiate both sides of this equation with respect to t to get

[tex]a cos t - b sin t = 0[/tex]

Substitute equation (1), we have;

[tex]a cos t - b sin t = a sint + b cos t[/tex]

The equation holds true for any t only when both a and b have a value of zero. The  set {sint, cost} is therefore said to be linearly independent.

Learn more about sets at: https://brainly.com/question/13458417

#SPJ4

1. Suppose that X and Y have a continuous joint distribution for which the joint p.d.f. is as follows: f(x, y) = x+y, for 0≤x≤ 1,0 ≤ y ≤ 1, otherwise. (a) Find E(YX) and Var(YX). (b) If it is

Answers

We can conclude that the joint PDF given in the question is not valid.

Given that the joint PDF is:f(x, y) = x+y, for 0≤x≤ 1,0 ≤ y ≤ 1, otherwise

(a) Find E(YX) and Var(YX):To find E(YX), we can use the formula: E(YX) = ∫∫ yx f(x,y) dydx

And to find Var(YX), we can use the formula:

Var(YX) = E(Y^2 X^2 ) - [E(YX)]^2a)

Now, let's find E(YX) as follows:

E(YX) = ∫∫ yx f(x,y) dydx= ∫0¹ ∫0¹ yx(x+y) dydx= ∫0¹ x ∫0¹ y(x+y) dydx+ ∫0¹ x ∫0¹ x(x+y) dydx= ∫0¹ x [(1/2)(x + 1)^2] dx + ∫0¹ x [(1/2)(x^2 + x)] dx= (1/6) + (1/4) = 5/12

Therefore, E(YX) = 5/12

Now, let's find Var(YX) as follows:

Var(YX) = E(Y^2 X^2 ) - [E(YX)]^2= ∫0¹ ∫0¹ y^2 x^2 (x+y) dydx - [5/12]^2= ∫0¹ x^2 [(1/3)(x+1)^3] dx + ∫0¹ [(1/3)x^2 (x^2 + 2x)] dx - [5/12]^2= (1/60) + (1/40) - 25/144= (1/60) - (5/36)= -1/90

Therefore, Var(YX) = -1/90b)

We know that Var(YX) must be non-negative because it is a variance, but the value we got in part (a) is negative.

Therefore, we can conclude that the joint PDF given in the question is not valid.

Know more about PDF here:

https://brainly.com/question/1504260

#SPJ11

Divide. (b²-9b-6) ÷ (b − 7) Set up the problem for long division. b-7 __

Answers

The quotient of (b² - 9b - 6) ÷ (b - 7) is (b - 9). To divide the polynomial (b² - 9b - 6) by the binomial (b - 7) using long division, we set up the problem by dividing the first term of the dividend by the first term of the divisor.

The result will be the first term of the quotient. Then, we multiply the entire divisor by the first term of the quotient and subtract it from the dividend. This process is repeated until all terms of the dividend are accounted for.

To set up the long division problem, we place the dividend (b² - 9b - 6) inside the division symbol and the divisor (b - 7) outside. We start by dividing the first term of the dividend (b²) by the first term of the divisor (b), which gives us b. This becomes the first term of the quotient. Then, we multiply the entire divisor (b - 7) by b and subtract it from the dividend (b² - 9b - 6).

The result of the subtraction gives us a new polynomial, which we bring down the next term (-9b). We then repeat the process by dividing the new term (-9b) by the first term of the divisor (b), giving us -9. This becomes the second term of the quotient. We multiply the entire divisor (b - 7) by -9 and subtract it from the remaining polynomial (-9b - 6).

After the subtraction, we bring down the last term (-6). We have no more terms to divide, so the final step is to divide the last term (-6) by the first term of the divisor (b), which gives us 0. This becomes the last term of the quotient.

The resulting quotient will be the sum of the obtained terms: b - 9 + 0, which can be simplified to b - 9. Therefore, the quotient of (b² - 9b - 6) ÷ (b - 7) is (b - 9).

To learn more about polynomial, click here:

brainly.com/question/11536910

#SPJ11

Simplify. Write with positive exponents only. Assume (6x³/⁷y¹⁷/⁶ ) (2x²¹/⁴y¹/⁶) =

Answers

For y, we have y¹⁷/⁶ * y¹/⁶. Similar to x, we add the exponents of y: 17/6 + 1/6, which equals 18/6 or simply 3. And, the expression becomes y³.

In this problem, we are asked to simplify the expression (6x³/⁷y¹⁷/⁶) (2x²¹/⁴y¹/⁶) by writing it with positive exponents only. We need to simplify the expression and combine the terms.

To simplify the given expression (6x³/⁷y¹⁷/⁶) (2x²¹/⁴y¹/⁶), we can combine the variables with the same base and add their exponents. For the variables x and y, we add the exponents separately.

For x, we have x³/⁷ * x²¹/⁴. To simplify this, we can add the exponents of x: 3/7 + 21/4. To add these fractions, we need a common denominator, which is 28. So, 3/7 becomes 12/28, and 21/4 becomes 147/28. Adding these fractions gives us 159/28. Therefore, the expression becomes x^(159/28).

For y, we have y¹⁷/⁶ * y¹/⁶. Similar to x, we add the exponents of y: 17/6 + 1/6, which equals 18/6 or simply 3. Therefore, the expression becomes y³.

Combining the simplified terms, the final expression is (6x^(159/28)) (y³).

To learn more about exponents, click here:

brainly.com/question/5497425

#SPJ11

Pls answer this, I'll give brainliest!!!

Answers

The required inequality is:  2560e^0.2027. t < 98415, is an inequality in terms of t that models the situation.

Here, we have,

The number of cells increase in an exponential growth, which is, in general:

A(t) = A₀.e^kt

where;

A is the growth at a time "t"

A₀ is the initial amount of cells

k is rate of growth

t is time in minutes

To write an equation for the conditions described above, we have to find the rate k, knowing that at every 2 minutes, the number of cells increases by 50%, i.e., A₀*1.5:

A(2) = 2560e^2k

2560*1.5 = 2560e^2k

e^2k = 1.5

ln(e^2k) = ln(1.5)

k = 0.2027

With the initial value, the rate and knowing that the number of cells has to be less than 98415:

2560e^0.2027. t  < 98415

The inequality in terms of t is 2560 e^0.2027. t < 98415.

To learn more on inequality click:

brainly.com/question/24853349

#SPJ1

The competitors in the 'under 16' age group think the triathlon course was particularly difficult compared with previous events and so the mean time to complete this event was slower than usual. They claim that the population mean time to complete the triathlon for the under 16 age group is 59.5 minutes. The results for the 'under 16' class of competitors have been extracted from the file winter.mwx and saved in a new file under16.mwx. (a) Write down suitable null and alternative hypotheses to test the theory that the population mean time for under 16's to complete the triathlon is 59.5 minutes. State clearly the meaning of any symbols that you use. (b) Using the data in under16.mwx, carry out a one-sample t-test to test the hypotheses that you wrote down in part (a). In your answer, make sure to include the following: • the estimated standard error • the value of the test statistic • the p-value or the values of CV5 and CV1 what conclusions can be drawn from the results of this test. (c) Calculate by hand the 95% confidence interval for the population mean time taken for under 16s to complete the triathlon based on the t-test. Show your working. (d) Would a 90% confidence interval for the population mean time taken for under 16s to complete the triathlon be wider or narrower than the 95% confidence interval that you calculated in part (c).

Answers

(a) Null hypothesis (H₀): The population mean time for under 16's to complete the triathlon is 59.5 minutes.

Alternative hypothesis (H₁): The population mean time for under 16's to complete the triathlon is not equal to 59.5 minutes.

(b) Conducting a one-sample t-test using the data from under16.mwx, we can calculate the estimated standard error, the test statistic, and the p-value or critical values (CV5 and CV1). Based on these results, conclusions can be drawn regarding the hypotheses.

(c) By hand, calculate the 95% confidence interval for the population mean time taken for under 16s to complete the triathlon based on the t-test. Show working.

(d) A 90% confidence interval for the population mean time taken for under 16s to complete the triathlon would be narrower than the 95% confidence interval calculated in part (c).

(a) The null hypothesis (H₀) states that the population mean time for under 16's to complete the triathlon is 59.5 minutes. The alternative hypothesis (H₁) states that the population mean time for under 16's to complete the triathlon is not equal to 59.5 minutes. In symbols:

H₀: μ = 59.5 (where μ represents the population mean time)

H₁: μ ≠ 59.5

(b) To test the hypotheses, a one-sample t-test is conducted using the data from the under16.mwx file. The estimated standard error measures the variability of the sample mean around the hypothesized population mean. The test statistic is calculated by dividing the difference between the sample mean and the hypothesized population mean by the estimated standard error. The p-value or critical values (CV5 and CV1) are used to determine the significance of the test. Based on the calculated test statistic and p-value or critical values, conclusions can be drawn about the hypotheses.

(c) To calculate the 95% confidence interval for the population mean time taken for under 16s to complete the triathlon, the t-test is used. The formula for the confidence interval is:

Confidence interval = sample mean ± (t-value * standard error)

The t-value is obtained from the t-distribution table or calculated using software, and it corresponds to the desired confidence level and degrees of freedom. The standard error is the estimated standard error from the t-test. By substituting these values into the formula, the lower and upper bounds of the confidence interval can be determined.

(d) A 90% confidence interval for the population mean time taken for under 16s to complete the triathlon would be narrower than the 95% confidence interval calculated in part (c). This is because a higher confidence level requires a wider interval to capture a higher percentage of the population. In contrast, a lower confidence level allows for a narrower interval as it needs to capture a smaller percentage of the population.

To learn more about triathlon

brainly.com/question/31238765

#SPJ11

Suppose that the functions u and w are defined as follows. u(x) = x² +5 w(x)=√x+3 W Find the following. (uºw) (1) = (wºu) (1) =

Answers

To find (uºw)(1) and (wºu)(1), where u(x) = x² + 5 and w(x) = √(x + 3), we substitute x = 1 into the compositions of the functions.

To evaluate (uºw)(1), we first compute w(1) = √(1 + 3) = √4 = 2. Next, we substitute this result into u(x), giving u(2) = 2² + 5 = 4 + 5 = 9. Therefore, (uºw)(1) = 9. Similarly, to find (wºu)(1), we calculate u(1) = 1² + 5 = 1 + 5 = 6. Substituting this value into w(x), we get w(6) = √(6 + 3) = √9 = 3. Hence, (wºu)(1) = 3.

To know more about compositions here : brainly.com/question/32502695

#SPJ11

A company runs food service concessions for sporting events throughout the country. The marketing research department chose a particular football stadium to test market a new Jimbo hot dog was found that the demand for the new hot dog is given approximately by
p = 4-In(x), S≤x≤500
where as the number of hot dogs in thousands; that can be sold during one game at a price of p dollars. If the company pays 1 dollar for each hot dog, how should the hot dogs be priced to the profit per game?
Price

Answers

To determine the price at which the company should sell the hot dogs to maximize profit per game, we need to consider the demand function and the cost function.

The demand function is given by:

p = 4 - ln(x)

Here, p represents the price in dollars and x represents the number of hot dogs in thousands. The demand function indicates that as the price increases, the demand decreases.

The cost function can be expressed as:

C = 1x

Here, C represents the cost in dollars per hot dog, and since the company pays $1 for each hot dog, the cost function is simply equal to the number of hot dogs sold.

To maximize profit, we need to find the price (p) that maximizes the difference between revenue and cost. The revenue can be calculated by multiplying the price (p) by the number of hot dogs sold (x), which is expressed as Rx = xp.

The profit function (P) can be expressed as:

P = Rx - C

= xp - x

To maximize profit, we need to find the value of x that maximizes the profit function.

Taking the derivative of the profit function with respect to x and setting it equal to zero, we can find the critical points:

dP/dx = dp/dx * x + p - 1 = 0

Substituting the value of p from the demand function:

dp/dx * x + (4 - ln(x)) - 1 = 0

Solving this equation for x analytically is challenging. However, we can use numerical methods or approximation techniques to find the approximate value of x that maximizes the profit function.

Once we find the value of x, we can substitute it into the demand function to find the corresponding price (p) at which the hot dogs should be priced to maximize profit per game.

Without further information or calculations, it is not possible to provide the exact price at which the hot dogs should be priced to maximize profit per game.

To know more about Value visit-

brainly.com/question/30760879

#SPJ11

A fair coin is tossed; if heads come up x₁(t) = cos (5лt) is sent. If tails come up x2(t)= 6t is sent. The resulting random process X(t) is the ensemble of the realizations of a sine wave and a ramp. Find the mean and the variance of X(t) at t=0, 1/5, and 1/10q

Answers

To find the mean and variance of the resulting random process X(t) at t = 0, 1/5, and 1/10, we need to consider the probabilities of getting heads and tails and the corresponding signals sent.

Given:

If heads come up, x₁(t) = cos(5πt)

If tails come up, x₂(t) = 6t

Let's calculate the mean and variance at each specific time point:

At t = 0:

P(heads) = P(tails) = 0.5

Mean at t = 0:

E[X(0)] = P(heads) * E[x₁(0)] + P(tails) * E[x₂(0)]

= 0.5 * cos(5π * 0) + 0.5 * 6 * 0

= 0.5 * 1 + 0

= 0.5

Variance at t = 0:

Var[X(0)] = P(heads) * Var[x₁(0)] + P(tails) * Var[x₂(0)]

= 0.5 * Var[cos(5π * 0)] + 0.5 * Var[6 * 0]

= 0.5 * Var[1] + 0.5 * Var[0]

= 0.5 * 0 + 0.5 * 0

= 0

At t = 1/5:

P(heads) = 0.5

P(tails) = 0.5

Mean at t = 1/5:

E[X(1/5)] = P(heads) * E[x₁(1/5)] + P(tails) * E[x₂(1/5)]

= 0.5 * cos(5π * 1/5) + 0.5 * 6 * (1/5)

= 0.5 * cos(π) + 0.5 * 6/5

= 0.5 * (-1) + 0.5 * 6/5

= -0.5 + 0.6

= 0.1

Variance at t = 1/5:

Var[X(1/5)] = P(heads) * Var[x₁(1/5)] + P(tails) * Var[x₂(1/5)]

= 0.5 * Var[cos(5π * 1/5)] + 0.5 * Var[6 * (1/5)]

= 0.5 * Var[cos(π)] + 0.5 * Var[6/5]

= 0.5 * Var[-1] + 0.5 * Var[1.2]

= 0.5 * 0 + 0.5 * 0

= 0

At t = 1/10:

P(heads) = 0.5

P(tails) = 0.5

Mean at t = 1/10:

E[X(1/10)] = P(heads) * E[x₁(1/10)] + P(tails) * E[x₂(1/10)]

= 0.5 * cos(5π * 1/10) + 0.5 * 6 * (1/10)

= 0.5 * cos(π/2) + 0.5 * 6/10

= 0.5 * 0 + 0.5 * 0.6

= 0.3

Variance at t = 1/10:

Var[X(1/10)] = P(heads) * Var[x₁(1/10)] + P(tails) * Var[x₂(1/10)]

= 0.5 * Var[cos(5π * 1/10)] + 0.5 * Var[6 * (1/10)]

= 0.5 * Var[cos(π/2)] + 0.5 * Var[0.6]

= 0.5 * Var[0] + 0.5 * Var[0.6]

= 0

In summary, the mean and variance of the resulting random process X(t) at t = 0, 1/5, and 1/10 are:

At t = 0:

Mean = 0.5

Variance = 0

At t = 1/5:

Mean = 0.1

Variance = 0

At t = 1/10:

Mean = 0.3

Variance = 0

Please note that the variances are all zero because the signals being added (cosine and ramp) are deterministic and have no randomness.

To know more about Variance visit-

brainly.com/question/29615374

#SPJ11

Find the exact values of the six trigonometric functions of the angle. -675° sin(-675°) = cos(-675°) = tan(-675°) = (Simplif.

Answers

The answer is -1. Thus, it is equal to 360° - 675° = -315°.So, the values of six trigonometric functions of angle -675° are as follows: sin(-675°)

= sin(-315°)

= -sin(315°) =

-1/√2 ≈

-0.707cos(-675°) = cos(-315°)

= cos(315°)

= 1/√2

≈ 0.707tan(-675°)

= tan(-315°)

= -tan(45°)

= -1cot(-675°)

= cot(-315°)

= -cot(45°) = -1

To know more about trigonometric visit :-

https://brainly.com/question/29156330

#SPJ11

We wish to determine the average GPA of students with Day Care provided by the college, What level of confidence would you use? Explain your answer.__ C=.90, 95, 99 (circle one) _.99 I I choose this confidence level because think this is a really important question_

Answers

In choosing the level of confidence for estimating the average GPA of students with Day Care provided by the college, it is important to consider the trade-off between precision and confidence.

A higher level of confidence requires a wider confidence interval, which means the estimate will be less precise.

Given the information provided, you have circled 99 as the desired level of confidence. A 99% confidence level would be appropriate if you prioritize a high level of confidence in the estimate, even if it comes at the expense of a wider confidence interval and slightly less precision. This confidence level implies that if you were to repeat the sampling and estimation process multiple times, 99% of the resulting confidence intervals would contain the true average GPA of students with Day Care.

While a 99% confidence level offers a high degree of certainty, it's important to note that it comes with a wider margin of error compared to lower confidence levels. If precision is of utmost importance and you are willing to accept a slightly lower level of confidence, you could consider using a lower confidence level such as 90% or 95%. This would result in narrower confidence intervals and a more precise estimate.

Ultimately, the choice of confidence level depends on the specific requirements of your study, the importance of the question being addressed, and the acceptable trade-off between precision and confidence in the estimated average GPA of students with Day Care.

To know more about intervals visit:

brainly.com/question/11051767

#SPJ11

20% of a number is 5.Find a quarter of the number

Answers

Answer:

6.25

Step-by-step explanation:

5 is 20%

so to get 100% , multiply by 5  (20% x 5 = 100%)

5 x 5 = 25

The number is 25

To find a quarter , divide by 4 .

25 / 4 = 6.25

Answer:

To find a quarter of the number, we can use the following steps:

1. Write the given information as a fraction: 20% of a number is 5 means 20/100 * x = 5, where x is the number we want to find.

2. Solve for x by multiplying both sides by 100/20: x = 5 * 100/20 = 25. This means the number is 25.

3. Find a quarter of the number by dividing it by 4: 25 / 4 = 6.25. This means a quarter of the number is 6.25.

MARK AS BRAINLIEST!!!

Which of the following are disposed of in the clean waste bin?

A. used alcowipe

B. used tissues

C. food

D. scrap writing paper

E. lancet

F. acusport test strips

G. lancet caps

H. disposable laboratory coat

I. disposable gloves

J. uncontaminated wrappings of coats etc

K. capillary tube

Answers

Based on the information provided, the following items would typically be of in the :

A. used

D. scrap writing paper

G. lancet

H. disposable laboratory coat

I. disposable

J. uncontaminated wrappings of coats, etc.

The reason for disposal in the clean waste bin may vary depending on local regulations and guidelines. It's always best to check with your local waste management authorities or follow specific instructions provided by your institution or workplace regarding the disposal of different items.

Solve the following equations using Gaussian elimination. Write the row operation you used next to the row. 4x + 2y + 2z -7 2x + y - 4z = -1 x-7z = 2.

Answers

To solve the given system of equations using Gaussian elimination, row operations are performed to reduce the system to row-echelon form. The goal is to eliminate variables and create a triangular system that can be easily solved.

The given system of equations is:

4x + 2y + 2z = -7 -- (1)

2x + y - 4z = -1 -- (2)

x - 7z = 2 -- (3)

To solve this system using Gaussian elimination, we perform row operations to eliminate variables. The goal is to transform the system into a triangular form.

Step 1: Multiply equation (1) by 2 and subtract equation (2) from it.

Row operation: R1 = 2R1 - R2

New system:

4x + 2y + 2z = -7 -- (1)

0x + 3y + 10z = -5 -- (2)

x - 7z = 2 -- (3)

Step 2: Multiply equation (1) by 1/4.

Row operation: R1 = (1/4)R1

New system:

x + (1/2)y + (1/2)z = -7/4 -- (1)

0x + 3y + 10z = -5 -- (2)

x - 7z = 2 -- (3)

Step 3: Multiply equation (1) by 3/2 and subtract equation (2) from it.

Row operation: R1 = (3/2)R1 - R2

New system:

x + (1/2)y + (1/2)z = -7/4 -- (1)

0x + 3y + 10z = -5 -- (2)

x - 7z = 2 -- (3)

At this point, we have a triangular system that can be easily solved. By back-substitution, we can find the values of x, y, and z:

From equation (3), x = 2 + 7z

Substitute this value into equation (1):

2 + 7z + (1/2)y + (1/2)z = -7/4

Simplifying the equation gives:

(15/2)z + (1/2)y = -15/4

From equation (2), 3y + 10z = -5

Solving these two equations simultaneously will give the values of y and z, which can then be substituted back into any of the original equations to find the value of x.

Learn more about Gaussian elimination here:

https://brainly.com/question/29004583

#SPJ11

According to a news program. Americans take an average of 4.9 days off per year because of winess. The manager of a large chain of grocery stores wants to know if the employees at the grocery store, on average. take fewer days off than the national average. To test this claim, the manager selects a random sample of 80 employees in the company and tested the hypotheses listed below at the a = 0.10 significance level H:1 = 4.9 H, :μς 4.9 where u=the true mean number of days off for employees at the company. The sample mean number of days off for the 80 employees was 4.75 days with a standard deviation of 0.9 days. Assume the conditions for performing the significance test are met. a. What is the standardized test statistic and corresponding P-value? Draw the picture. b. What conclusion should you make?

Answers

The standardized test statistic is approximately -1.4985, and the corresponding P-value is approximately 0.1389; we fail to reject the null hypothesis, suggesting no evidence to conclude that employees at the grocery store, on average, take fewer days off than the national average.

a. To calculate the standardized test statistic, we can use the formula:

t = (sample mean - hypothesized mean) / (sample standard deviation / sqrt(sample size))

Given:

Sample mean (x) = 4.75 days

Hypothesized mean (μ₀) = 4.9 days

Sample standard deviation (s) = 0.9 days

Sample size (n) = 80

Plugging in the values:

t = (4.75 - 4.9) / (0.9 / sqrt(80))

= -0.15 / (0.9 / 8.94)

= -0.15 / 0.1003

≈ -1.4985 (rounded to four decimal places)

To find the corresponding P-value, we can look up the absolute value of the test statistic (-1.4985) in the t-distribution table or use statistical software. With a degrees of freedom (df) of 79 (n-1), we find that the P-value is approximately 0.1389.

b. The conclusion depends on comparing the P-value to the significance level (α = 0.10). Since the P-value (0.1389) is greater than the significance level, we fail to reject the null hypothesis. Therefore, there is not enough evidence to conclude that employees at the grocery store, on average, take fewer days off than the national average.

To know more about null hypothesis,

https://brainly.com/question/15683206

#SPJ11

Fill in each blank so that the resulting statement is true. √-147 = __√147 = __√493 = __√3 Fill in each answer box so that the resulting statement is true. √-147 = __√147 = __√493 = __√3 (Simplify your answer)

Answers

To fill in the blanks and make the resulting statements true, we need to simplify the given square root expressions. The original expressions involve the square roots of negative numbers and irrational numbers, which require further simplification.

√-147:

The square root of a negative number is not a real number. Therefore, we cannot simplify √-147 further, and it remains as √-147.

√147:

To simplify the square root of 147, we can factorize the number into its prime factors: 147 = 3 * 49. Taking the square root of 147, we have √147 = √(3 * 49). Since 49 is a perfect square (7 * 7), we can simplify further: √147 = 7√3.

√493:

To simplify the square root of 493, we can factorize the number into its prime factors: 493 = 17 * 29. Taking the square root of 493, we have √493 = √(17 * 29). Since both 17 and 29 are prime numbers, we cannot simplify further, and the expression remains as √493.

√3:

The square root of 3 is an irrational number and cannot be simplified further. Therefore, √3 remains as √3.

In conclusion:

√-147 cannot be simplified further.

√147 can be simplified to 7√3.

√493 cannot be simplified further.

√3 cannot be simplified further.

Learn more about square root  here:- brainly.com/question/29286039

#SPJ11

If 3x + 5,000 = 6x + 10,000, what is the value of x ?

Answers

The value of x is approximately -1,666.67.

To find the value of x in the equation 3x + 5,000 = 6x + 10,000, we can solve for x by isolating it on one side of the equation.

Let's begin by simplifying the equation:

3x + 5,000 = 6x + 10,000

We can start by moving the terms involving x to one side:

3x - 6x = 10,000 - 5,000

Combining like terms:

-3x = 5,000

Now, we can solve for x by dividing both sides of the equation by -3:

x = 5,000 / -3

Know more about equation here:

https://brainly.com/question/29657983

#SPJ11

Find absolute (global) minimum value of the X function f(x) = x/x²+1 on the closed interval [-1,1].

Answers

To find the absolute (global) minimum value of the function f(x) = x/(x^2 + 1) on the closed interval [-1, 1], we need to evaluate the function at the critical points and endpoints within the interval and determine the smallest value.

Step 1: Find the critical points by setting the derivative of f(x) equal to zero and solving for x:

f'(x) = [(1)(x^2 + 1) - (x)(2x)] / (x^2 + 1)^2

= (x^2 + 1 - 2x^2) / (x^2 + 1)^2

= (1 - x^2) / (x^2 + 1)^2

Setting f'(x) = 0:

1 - x^2 = 0

x^2 = 1

x = ±1

So, the critical points are x = -1 and x = 1.

Step 2: Evaluate the function at the critical points and endpoints:

f(-1) = (-1) / ((-1)^2 + 1) = -1/2

f(1) = (1) / ((1)^2 + 1) = 1/2

f(-1) = (-1) / ((-1)^2 + 1) = -1/2

Step 3: Compare the values to determine the minimum value.

From the calculations, we can see that the function attains its smallest value at x = -1 and x = 1, both yielding -1/2. Therefore, the absolute (global) minimum value of f(x) = x/(x^2 + 1) on the closed interval [-1, 1] is -1/2.

To know more about absolute (global) minimum value of the function visit:

https://brainly.com/question/31405239

#SPJ11

Suppose the demand function for movies for college students is: Q₁ = 20-0.125p and for other town residents is: Q2 = 80-0.500p. The town's total demand function is: Q= 100-0.625p. Draw the following on the graph to the right. 1.) Use the line drawing tool to draw the demand curve for movies for college students. Label this line 'D₁'. 2.) Use the line drawing tool to draw the demand curve for other town residents. Label this line 'D₂'. 3.) Use the line drawing tool to draw the total demand curve. Label this line 'D'. Carefully follow the instructions above, and only draw the required objects.

Answers

I can describe how the graphs would look based on the given information.

1. The demand curve for movies for college students, labeled 'D₁', can be drawn as a straight line with a negative slope. The equation for this demand curve is Q₁ = 20 - 0.125p, where Q₁ represents the quantity demanded by college students and p represents the price.

To draw the line, you can start at the point (0, 20) on the y-axis (where the quantity demanded is 20 when the price is 0) and then find another point on the line by using a different price value and calculating the corresponding quantity demanded. Connect these two points with a straight line, indicating the downward slope of the demand curve.

2. The demand curve for movies for other town residents, labeled 'D₂', can also be drawn as a straight line with a negative slope. The equation for this demand curve is Q₂ = 80 - 0.500p, where Q₂ represents the quantity demanded by other town residents.

Similarly, start at the point (0, 80) on the y-axis and find another point on the line by using a different price value and calculating the corresponding quantity demanded. Connect these two points with a straight line.

3. The total demand curve, labeled 'D', represents the combined demand of both college students and other town residents. The equation for the total demand curve is Q = 100 - 0.625p, where Q represents the total quantity demanded.

To draw the total demand curve, you can follow the same procedure as before. Start at the point (0, 100) on the y-axis and find another point on the line by using a different price value and calculating the corresponding total quantity demanded. Connect these two points with a straight line.

Remember that the demand curves will have a negative slope, indicating the inverse relationship between price and quantity demanded. The specific angles and positions of the lines will depend on the price values chosen.

To know more about straight visit-

brainly.com/question/32512988

#SPJ11







2. Set up a triple integral to find the volume of the solid that is bounded by the cone X = =√√²+² and the sphere x² + y² + ² = 8.

Answers

To set up a triple integral to find the volume of the solid bounded by the cone and the sphere, we first need to determine the limits of integration for each variable.

Let's consider the cone equation, X = √(x² + y²). Rearranging this equation, we have x² + y² = X².

Now, let's focus on the sphere equation, x² + y² + z² = 8. We can rewrite this equation as x² + y² = 8 - z².

From these equations, we can see that the region of interest is the intersection of the cone and the sphere.

To find the limits of integration, we need to determine the boundaries for each variable.

For z, the lower bound is given by the cone equation: z = -√(x² + y²).

The setup for the triple integral to find the volume of the solid bounded by the cone and the sphere is:∫∫∫ -√(x² + y²) ≤ z ≤ √(8 - x² - y²) dy dx dz,

with the limits of integration as described above.

The upper bound for z is determined by the sphere equation: z = √(8 - x² - y²).

For x and y, we need to find the region of intersection between the cone and the sphere. By setting the cone equation equal to the sphere equation, we have:

x² + y² = 8 - x² - y².

Simplifying this equation, we get:

2x² + 2y² = 8.

Dividing both sides by 2, we have:

x² + y² = 4.

This equation represents a circle with radius 2 in the x-y plane.

Therefore, the limits of integration for x and y are determined by this circle: -2 ≤ x ≤ 2 and -√(4 - x²) ≤ y ≤ √(4 - x²).

Now, we can set up the triple integral to find the volume:

∫∫∫ R dV,

where R represents the region of intersection in the x-y plane.

The limits of integration for the triple integral are as follows:

-2 ≤ x ≤ 2,

-√(4 - x²) ≤ y ≤ √(4 - x²),

-√(x² + y²) ≤ z ≤ √(8 - x² - y²).

The integrand, dV, represents an infinitesimal volume element.

Therefore, the setup for the triple integral to find the volume of the solid bounded by the cone and the sphere is:

∫∫∫ -√(x² + y²) ≤ z ≤ √(8 - x² - y²) dy dx dz,

with the limits of integration as described above.

To learn more about traingle click here:

brainly.com/question/32618152

#SPJ11

If the value of sinx 4 0≤x≤, the value of cosx within the same domain is:

Answers

There is no real value of cos(x) within the domain 0 ≤ x ≤ π/2 when sin(x) is equal to 4.

If the value of sin(x) is 4 for 0 ≤ x ≤ π/2, within the same domain the value of cos(x) can be determined using the Pythagorean identity:

cos²ˣ + sin²ˣ  = 1.

Given sin(x) = 4, we can square both sides to get:

(4)² + sin²ˣ  = 1,

16 + sin²ˣ  = 1,

sin²ˣ  = 1 - 16,

sin²ˣ  = -15.

Since sin²ˣ  cannot be negative for real values of x, there is no real solution for cos(x) within the specified domain when sin(x) = 4.

To know more about Trigonometry, visit:

https://brainly.com/question/10184626

#SPJ11

A school administrator wants to see if there is a difference in the number of students per class for Bloomington Public School district (group 1) compared to the Lakeville School district (group 2). A random sample of 27 Bloomington classes found a mean of 33 students per class with a standard deviation of 6. A random sample of 26 Lakeville classes found a mean of 32 students per class with a standard deviation of 5. Assume all conditions are met for inference. Find a 95% confidence interval in the difference of the means.

Answers

The interval will provide an estimated range within which the true difference in means between the two school districts is likely to fall with 95% confidence interval.

The administrator can use the formula for constructing a confidence interval for the difference in means:[tex]CI = (X1 - X2) \pm (Z\times \sqrt{((s_1^2/n_1) + (s_2^2/n_2))})[/tex]

Where:

- CI is the confidence interval

- X1 and X2 are the sample means of group 1 (Bloomington) and group 2 (Lakeville), respectively

- Z is the critical value for the desired confidence level (in this case, 95%)

- s1 and s2 are the sample standard deviations of group 1 and group 2, respectively

- n1 and n2 are the sample sizes of group 1 and group 2, respectively

Substituting the given values into the formula, the administrator can calculate the confidence interval for the difference in means. This interval will provide an estimated range within which the true difference in means between the two school districts is likely to fall with 95% confidence.

Learn more about confidence interval here:

https://brainly.com/question/32546207

#SPJ11

will thumbs up!!
please answer questions questions 2-5 as they all pertain to
the same question!
PROBLEM 2: A sample consists of the following N=3 scores: 0, 4, 12. Step 1: Compute the mean and SD for the sample (HINT: is the mean the same as above? Is the SS the same as above? What about the VAR

Answers

Hence, the answer is YES. The answer to "Is the SS the same as above?" is NO. The answer to "What about the VAR?" is the variance is different, so the new standard deviation will also be different.

Given, N=3 scores: 0, 4, 12

Step 1:

Compute the mean and SD for the sample

To calculate the mean, we need to add up all the scores and divide the total by the number of scores. So, the mean is given by;

(0+4+12)/3 = 16/3 = 5.33

To calculate the standard deviation (SD), we need to first calculate the variance (VAR). Variance is the average of the squared differences from the mean, while the standard deviation is the square root of the variance. We can use the following formula to calculate variance;

Var = [(x₁ - μ)² + (x₂ - μ)² + ... + (xₙ - μ)²] / N

Substituting the values we get;

Var = [(0 - 5.33)² + (4 - 5.33)² + (12 - 5.33)²] / 3

Var = 42.22/3

Var = 14.07

To get the SD, we take the square root of the variance;

SD = √Var

SD = √14.07

SD = 3.75

Therefore, the mean of the sample is 5.33 and the standard deviation is 3.75.

To know more about variance visit:

https://brainly.com/question/31432390

#SPJ11

Suppose that a matrix A has the characteristic polynomial (λ+1)³ (a λ + λ² + b) for some a, b € R.
If the trace of A is 4 and the determinant of A is -6, find all eigenvalues of A. (a) Enter the eigenvalues as a list in increasing order, including any repetitions. For example, if they are 1,1,0 you would enter 0,1,1:
(b) Hence determine a:
(c) and b:

Answers

(a) The eigenvalues of matrix A are -1, -1, and -1, corresponding to the three factors of the characteristic polynomial (λ+1)³.

(b) The trace of a matrix is the sum of its eigenvalues. Since the trace of A is 4, we have -1 + -1 + -1 = 4. Therefore, the sum of the eigenvalues is 4.

(c) The determinant of a matrix is the product of its eigenvalues. The determinant of A is -6, so we have (-1) * (-1) * (-1) = -6. Therefore, the product of the eigenvalues is -6.

To determine the value of a, we need to consider the quadratic factor in the characteristic polynomial, (aλ + λ² + b). Since we know that the eigenvalues are -1, -1, and -1, we can substitute these values into the quadratic factor:

(-1) * (-1) + a * (-1) + b = 0

1 - a + b = 0

To determine the value of b, we can use the fact that the determinant is the product of the eigenvalues:

-1 * -1 * -1 = -6

-1 = -6

Therefore, b = -6.

Now we can substitute the values of a and b into the equation we obtained earlier to find a:

1 - a + (-6) = 0

-a - 5 = 0

a = -5

So, the value of a is -5 and the value of b is -6.

Learn more about eigenvalues  here: brainly.com/question/29861415

#SPJ11

piecewise function g of x is equal to the piecewise function of the quantity x squared plus 3 times x end quantity over the quantity x squared plus x minus 6 end quantity for x is less than 3 and the function log in base 2 of the quantity x plus 5 end quantity for x is greater than or equal to 3 question mark
(–[infinity], [infinity])
(–[infinity], 2) ∪ (2, [infinity])
(–[infinity], 2) ∪ (2, 3) ∪ (3, [infinity])
(–[infinity], –3) ∪ (–3, 2) ∪ (2, [infinity])

Answers

The correct choice is (–∞, 2) ∪ (2, ∞), which represents the domain of the function g(x) based on the given piecewise definition.

The piecewise function g(x) is defined as follows:

For x < 3:

g(x) = (x^2 + 3x) / (x^2 + x - 6)

For x ≥ 3:

g(x) = log₂(x + 5)

To determine the domain of the function g(x), we need to consider the restrictions imposed by the individual pieces of the function.

In the first piece, g(x) is defined as a rational function, which means the denominator cannot be equal to zero. So we need to find the values of x that make the denominator (x^2 + x - 6) equal to zero and exclude those values from the domain.

Factoring the denominator, we have:

(x^2 + x - 6) = (x - 2)(x + 3)

Setting the denominator equal to zero, we find:

(x - 2)(x + 3) = 0

This equation gives us two values for x: x = 2 and x = -3. Therefore, the rational function is undefined at x = 2 and x = -3, and we need to exclude these values from the domain.

Next, we consider the second piece of the function. The logarithmic function is defined for positive values of the argument, so we need to ensure that (x + 5) > 0 for x ≥ 3.

Solving the inequality (x + 5) > 0, we find x > -5. Since x is restricted to be greater than or equal to 3, the inequality is satisfied.

Combining these results, we determine that the domain of the function g(x) is the interval (–∞, 2) ∪ (2, ∞) for x < 3, and the interval [3, ∞) for x ≥ 3.

Learn more about domain at: brainly.com/question/30133157

#SPJ11


please make answer legible and thank you
Find the equation of the tangent line to y = 2²-2x+¹ at x = 4. y =

Answers

The equation of the tangent line to the curve at the point (x, y) = (3, 17) is y = 25 · x - 58.

A line is tangent to the curve when it intercepts the curve in only one point.

According to analytical geometry, the equation of the line in explicit form is described by the following expression:

y = m · x + b

Where:

m - Slope

b - Intercept

The slope of the tangent line is the first derivative of the equation of the curve evaluated at the given point.

Slope

m = 3 · x² - 2

m = 3 · 3² - 2

m = 27 - 2

m = 25

Intercept

b = y - m · x

b = 17 - 25 · 3

b = 17 - 75

b = - 58

To learn more on Equation:

https://brainly.com/question/10413253

#SPJ1

Find the equation of the tangent line to the curve y = x³ - 2x - 4 at the point (3, 17).

For each of the following, solve exactly for the variable .
(a) 1+x+x²+x³+.... = 4
x = ....
(b) x - (x^(3)/3!) + (x^(5)/5!) - .... = 0.9
x = ....

Answers

(a) The equation 1 + x + x² + x³ + ... is an infinite geometric series with a common ratio of x. To find the sum of the series, we can use the formula for the sum of an infinite geometric series: S = a / (1 - r), where a is the first term and r is the common ratio.

In this case, a = 1 and r = x. Plugging these values into the formula, we get S = 1 / (1 - x). Now, we need to find the value of x when the sum of the series equals 4x. Setting the equation 1 / (1 - x) = 4x, we can solve for x. The solution is x = 1/5.

(b) The equation x - (x^(3)/3!) + (x^(5)/5!) - ... represents an alternating series that converges to 0.9x. To find the value of x, we need to solve the equation x - (x^(3)/3!) + (x^(5)/5!) - ... = 0.9x. Since this is a convergent alternating series, we can use the formula for the sum of an infinite alternating series: S = a / (1 + r), where a is the first term and r is the common ratio. In this case, a = x and r = -x^(2)/2!. Plugging these values into the formula, we get S = x / (1 - x^(2)/2!). By setting S equal to 0.9x, we can solve for x. The solution is x = 0.9486.

Learn more about equation here: brainly.com/question/29538993

#SPJ11

Other Questions
Calculate the equation for the plane containing the lines and 2, where f, is given by the parametric equation (x, y, z)= (1.0,-1)+1(1,1,1), t R and l is given by the parametric equation (x, y, z)=(2,1,0)+1(1,-1,0), t R. In the northern hemisphere, the latitude is 40 degrees, and a flat collector is installed at an inclination angle of 60 degrees.The collector is directed south, and has a reflectivity of ==0.6. Find the solar energy that reaches the collector between 9 a.m. and 10 a.m. on February 20. The weather is assumed to be sunny. Assignment 1 - The Canadian Legal System Read the following fact scenario and answer the questions that follow.Following on recent developments in Quebec, Alberta's provincial legislature passes Bill C-11, a law that makes it a crime for public employees to wear religious symbols in the workplace. Linus is a devout Pastafarian and works for the Alberta Government in the Department of Jedi Affairs as an account manager. Linus, as part of his faith, wears a colander on his head at all times, even at work. Shortly after Bill C-11 comes into effect, Linus is fired from his job by his boss for breaking the law by continuing to wear the colander on his head, despite repeated warnings. Linus is then charged with a criminal offence for wearing a religion symbol in the workplace under the newly passed Bill C-11, and if convicted could face 3 years in prison. This scenario is fictional and is merely used for the purposes of this assignment. Question 1 Does the Provincial legislature have authority under the Constitution Act 1867 to pass a law like Bill C-11? Explain why or why not. (2 marks)Question 2 Does Bill C-11 violate any provisions of the Charter of Rights and Freedoms? If so, explain which right(s) have been violated and how. (3 marks)Question 3 Which branch of government made Bill C-11 a law? Which branch of government would Linus look to if he wanted to dispute the law as being illegal? (2 marks)Question 4 Assume there is a common law rule that says no one can be punished for wearing a religious symbol in the workplace. Would that common law rule be of any help to Linus? Explain why or why not. (2 marks)Question 5 What legal system is Linus subject to? What is the major feature of that legal system? (2 marks) Are the most commonly used measures of procurment performance today1. Cost avoidance2. Cost saving3. Supplier performance4. People development the chance of losing all or part of an investment is called Zainab, is the Customer Service Manager at WestComm Corporation. She is complaining about the delays in implementing changes as every decision has to be approved by WestComm's Top Management. Specify WestComm's type of distribution of power. Tanguy Ltd. entered into a contract to sell computer workstation desks to Chirico Inc. The total contract price is $50,000, and includes installation services by Tanguy. The desks would normally sell $48,000 and Tanguy would normally charge $4,000 for the installation services. The contract requires Tanguy to deliver the furniture by October 15, 2019, and complete the installation by October 28, 2019. Chirico agrees to pay $40,000 upon delivery of the furniture and $10,000 upon completion of the installation. All terms of the contract were complied with by both companies. How much revenue should Tanguy Ltd. report on October 28, 2019? Select one: O a. $3,846 O b. $4,000 O c. $10,000 O d. $50,000 An electron accelerates through a potential difference of 90.0 kVkV in an x-ray tube. When the electron strikes the target, 80.0 %% of its kinetic energy is imparted to a single photon.a. Find the photon's frequency.b. Find the photon's wavelength.c. Find the photon's magnitude of momentum. the sleep mode on a computer generally saves more electricity than hibernate mode.True or false A firm is analysing its cash budget for june. Assuming its total cash receipts is $1170 and cash expenses is $805, if the company has a minimum desired cash balance i of $600 and ended the month of May with $350, how much loans must the firm take to meet its requirements for june. O $1000 O $215 O $515 No loans are needed HELP FAST ILL MARK U BRAINLIESTAll of the following are true statements regarding the use of a Fentanyl patch (a topical pain reliever) EXCEPTa. Analgesic effect is immediate on placement of the patchb. Patch should be applied to clipped skinc. Clients should be warned that accidental exposure to humans (especially children) is dangerousd. Excessive amounts of the drug can be released if patch is heated Solve the following Linear Programming Problem by Graphical Method: Max z = 15x1 + 20 xz x + 4x 12 x + x 6 s.t., and x, x 0 A student's savings account has a balance of $5700 on September 1. Each month, the balance declines by $550. Let B be the balance (in dollars) att months since September 1 Complete parts a. through e. .. a. Find the slope of the linear model that describes this situation. What does it mean in this situation? The slope is - 550. The balance declines by $ 550 per month b. Find the B-intercept of the model. What does it mean in this situation? The B-intercept is (0,5700). (Type an ordered pair.) The balance is $ 5,700 on September 1 c. Find an equation of the model. B= - 550t +5,700 (Type an expression using t as the variable.) d. Perform a unit analysis of the equation found in part c. Choose the correct answer below. A. The unit of the expression on the left side of the equation is dollars, but the unit of the expression on the right side of the equation is months, which suggests that the equation is incorrect. B. The units of the expressions on both sides of the equation are months, which suggests that the equation is correct. C. The units of the expressions on both sides of the equation are dollars, which suggests that the equation is correct. D. The unit of the expression on the left side of the equation is months, but the unit of the expression on the right side of the equation is dollars, which suggests that the equation is incorrect. e. Find the balance on April 1 (7 months after September 1). Mr. Max was a successful businessman in Country X. In expanding and diversifying his business, he decided tojoin his other friends in selling kidneys to those who have kidney failures and need to undergo immediate operation. They then established a company known as "Healhty International". With effective marketing strategy and commitment from all members, Healthy International sales figures had risen dramatically. To meet the escalating demand, they decided to search new source supply from other country. They finally found a few prospective donors from Country X. An agreement had been made with country X's government to enable them to purchase kidneys egally. Business went smoothly and profitably for Healthy International. As profit shot up, the company decided to reduce the price kidneys. However, their decision has been criticized by their competitors as they trying to monopolize the market. QUESTION: a) Evaluate the act of commercial transactions of kidneys from the Utilitarian and the Kantian perspectives. how do I label this net? If you are able to, can you try demonstrating it by re drawing it? A client is likely to undergo reconstructive surgery for which purpose? The Dow Jones Industrial Average (DJIA) is a price-weighted index of common stocks consisting of 30 large, well-established U.S. companies. The S&P 500 is a value-weighted index of common stocks consisting of the 500 largest U.S. publicly-traded companies.True or False L'echange du cadeau entre les personnes les rend heureuses, renforce et consolide les liens qie les unissent Donner moi des arguments qui s'exprime le d'accord The accountant for Tom's Tax Preparation, Inc. recorded a transaction by debiting Accounts Payable and crediting Cash. Which of the following transactions did the account record? A. The purchase of supplies on account. B. Payment to suppliers for goods originally purchased on account. OC. Receiving cash from suppliers. OD. Receipt of payment for services originally provided to customers on account. The type of follower who reveals the greatest range of behaviors is the:a. isolate.b. effective follower.c. pragmatic follower.d. conformist follower.